Can you prove E_{ij}AE_{kl}=a_{jk}E_{il} with the given formula for A?

  • Thread starter Thread starter lesliemath
  • Start date Start date
  • Tags Tags
    Matrix
Click For Summary
To prove E_{ij}AE_{kl}=a_{jk}E_{il}, begin by substituting the provided formula for matrix A into the expression. Focus on the properties of the elementary matrices E_{ij} and E_{kl}, which manipulate specific rows and columns of A. By applying these properties, you can simplify the expression step by step. It’s crucial to understand how these matrices interact with A to derive the desired result. This approach will guide you toward completing the proof effectively.
lesliemath
Messages
1
Reaction score
0
matrix -- proving need help!

please help i have no idea how to do this question. please give me some hints :(
 

Attachments

  • math.jpg
    math.jpg
    40.8 KB · Views: 409
Physics news on Phys.org


Part (a) is asking you to show that E_{ij}AE_{kl}=a_{jk}E_{il}, and it's giving you a formula for A. So why don't you start by writing E_{ij}AE_{kl}=, and then use the formula for A that you were given?
 
Last edited:
Question: A clock's minute hand has length 4 and its hour hand has length 3. What is the distance between the tips at the moment when it is increasing most rapidly?(Putnam Exam Question) Answer: Making assumption that both the hands moves at constant angular velocities, the answer is ## \sqrt{7} .## But don't you think this assumption is somewhat doubtful and wrong?

Similar threads

Replies
9
Views
2K
  • · Replies 25 ·
Replies
25
Views
3K
Replies
5
Views
2K
  • · Replies 3 ·
Replies
3
Views
1K
  • · Replies 1 ·
Replies
1
Views
2K
Replies
1
Views
1K
Replies
2
Views
2K
  • · Replies 11 ·
Replies
11
Views
2K
  • · Replies 10 ·
Replies
10
Views
3K
  • · Replies 4 ·
Replies
4
Views
2K